Science, asked by Chhayadahiya9559, 1 year ago

If angular momentum is conserved in a system whose moment of inertia is decreased will its rotational energy be conserved

Answers

Answered by MacTavish343
6
¤¤¤ROTATIONAL¤¤¤

L is angular momentum.
I is moment of Inertia
w is angular momentum.

L= Iw
then,

I1w1 = I2w2
or I1w1/I2w2 = 1  
Now  Rotaional Kinetic energy 

=1/2 I1​w1​2 / 1/2 I2 w22  
=I1w12I2w22 = w1w2/ ≠ 1 
so ​rotational kinetic energy not conserved

bunny122: plz explain it
Similar questions